Coefficients binomiaux congrus modulo $p^2$

Bonjour,
Je cherche une démonstration de : $\forall p \text{ premier}, \forall (n,m)\in\mathbb{N}^2, \displaystyle\binom{pm}{pn} \equiv \binom{m}n \, [p^2]$. J'ai essayé de mettre au même dénominateur $\displaystyle\binom{pm}{pn} - \binom{m}n = \frac{ \color{red}{(pm)!} - \color{green}{ m! \frac{(pn)!}{n!}\frac{(pm-pn)!}{(m-n)!} } }{ \color{blue}{(pn)!(pm-pn)!} } = \frac{\color{red}{A}-\color{green}{B}}{\color{blue}{C}}$ et de calculer les valuations $p$-adiques de $A,B,C$ mais elles sont parfois égales. Et je ne vois pas de technique par identification de coefficients de polynômes sur $\mathbb{Z}/p^2\mathbb{Z}$ (comme on pourrait faire si on était seulement modulo $p$).
Merci d'avance.

Réponses

  • Bonjour,

    Récurrence. As-tu essayé ?
  • C'est lié au théorème de Wolstenholme, me semble-t-il, et vrai mod. $p^3$ pour $p$ premier, $ p>3$
  • Bonjour,

    Avec des notations idoines, par récurrence :
    $\displaystyle \binom{p m}{p n} =\binom{m}{n}\pmod{p^2}.$

    Pour $m=0$ et $\forall n \leq m$, $n=0$ et $\displaystyle 1=1\pmod{p^2}.$

    On a $\displaystyle \binom{pm+p}{p n}=\binom{pm}{p n}+\sum_{j=1}^{p-1} \binom{pm }{p n-j} \binom{p}{j}+\binom{pm}{pn-p}$, qu’on aura plaisir à redémontrer.
    Le premier terme dans la somme vaut $$\displaystyle \binom{m}{n}\pmod{p^2}$$ par hypothèse de récurrence.
    Le second terme vaut $$\displaystyle 0\pmod{p^2},$$qu’on démontrera avec aisance.
    Le troisième terme vaut $$\displaystyle \binom{m}{n-1}\pmod{p^2}$$ par hypothèse de récurrence (dite forte sur $n$).
    On a donc $$\displaystyle \binom{pm+p}{p n}=\binom{m}{n}+\binom{m}{n-1}\pmod{p^2}.$$
    Voilà !
  • Effectivement ; je n'avais pas pensé à Vandermonde ! Merci beaucoup !
  • Pour le deuxième terme, je vois bien que $ \displaystyle \binom{p}{j} \equiv 0 \pmod{p}$ (et non $ \pmod{p^2}$), mais l'aisance me manque pour en déduire que ce deuxième terme tout entier est $ \equiv 0 \pmod{p^2}$.
    Bonne journée.
    Fr. Ch.
  • La valuation $p$-adique de $\binom{pm}{pn-j}$ est $v_p((pm)!)-v_p((pn)!-j)-v_p([p(m-n)+j]!)= m-(n-1)-(m-n)=1$ car $1\leqslant j\leqslant p-1$.

    Édit : J'ai dit une connerie. Merci Chaurien. Morale : ne pas poster au réveil.
  • @ Calli
    Affirmes-tu que $v_p((pm)!)=m$ ? Ça ne me semble pas vrai en général.
    Et $v_p((pn)!-j)=n-1$, non plus.
  • Je conseille de jeter un coup d’œil ici : https://en.wikipedia.org/wiki/Wolstenholme's_theorem
  • Bonjour,

    @chaurien : l'aisance vient de la relation $\displaystyle \binom{pm+a}{pn+b} = \binom{m}{n}\binom{a}{b} \pmod{p}.$ Pour monter par l'escalier au deuxième étage, on commence par le premier.
    Et alors chacun des termes du produit en question est nul modulo p.
  • @Yves M.
    La facile congruence que tu exhibes est fausse,et se révèle donc difficile à démontrer, surtout dans le cas où $p=5,\:m=2,\: n=1,\: a=7,b=1:$
    $$ \displaystyle \binom {pm+a}{pn+b}= \binom {17}6 \equiv 1\:\:\text{ et}\: \: \displaystyle \binom mn \binom ab = \binom 21 \binom 71 \equiv 4 \mod 5.$$
    Bien entendu, en rajoutant des hypothèses sur $a,b,m,n,\:$ on peut se débrouiller pour la rendre correcte.

    Cela dit,l'implication $ j\in [\![1;p-1]\!] \implies \displaystyle \binom {pm}{pn-j}\equiv 0 \mod p $ se justifie par exemple avec: $\:\:pm\displaystyle \binom{pm-1}{pn-j-1} = (pn-j)\binom {pm}{pn-j}$
    Donc: $\quad p \:\:\text{divise} \: \displaystyle(pn-j) \binom {pm}{pn-j} \quad\:\text{et}\:\quad p\wedge (np-j )=1,\quad$ ce qui entraine: $\forall j\in [\![1;p-1]\!],\: \displaystyle \binom {pm}{pn-j} \binom pj \equiv 0 \mod p^2.$
  • Peut-être la formule donnée par YvesM est-elle bonne si $a,b,m,n $ sont plus petits que $p$, selon un beau théorème d’Édouard Lucas :
    https://en.wikipedia.org/wiki/Lucas's_theorem
    Bonne soirée.
    Fr. Ch.
  • Bonjour,

    Oui, je ne m’embarrasse pas de définir les notations. La démonstration de @LOU 16 rend la récurrence encore plus facile.
  • Ma préférence va à la solution la plus simple, celle de LOU16.
    La formule $\: \displaystyle n \binom mn =m \binom {m-1}{n-1}$, nonobstant sa simplicité, a de nombreuses applications, et pas seulement en Arithmétique.
    Mais en fait on a : $\displaystyle \binom{p m}{p n} \equiv \binom{m}{n}\pmod{p^3}$ pour $p$ premier, $p \ge 5$, et la démonstration ci-dessus ne permet pas d'obtenir ce résultat.
    Bonne soirée.
    Fr. Ch.
  • Re,
    Pour la congruence $\mod p^3\quad (p\geqslant 5)$, je propose cette preuve qui fait appel, comme l'a dit Chaurien, au théorème de Wolstenholme.

    Notation : $\: \forall a \in \Z$ tel que $a\not\equiv 0 \mod p, \: \widehat a$ désigne l'unique entier tel que $a \hat a \equiv 1 \mod p\:\:$ et $\:\:1\leqslant a \leqslant p-1.$
    Il est alors commode de se placer dans l'anneau $\mathbb A =\{ \dfrac ab \in \Q\mid a,b \in \Z,\: b\wedge p = 1 \}$ et d'y définir la relation de congruence $\equiv$ par:
    $$\:\:\forall x,y \in\mathbb A,\: \forall n \in \N^*,\quad x\equiv y \mod p^n \mathbb A \iff x-y \in p^n\mathbb A.$$ Cette relation, compatible avec les opérations habituelles, l'est aussi avec la division, c'est à dire:
    $$x\equiv x',\: y\equiv y' \mod p^n\mathbb A,\quad y\not\equiv 0 \mod p \mathbb A \implies \dfrac xy =\dfrac {x'}{y'} \mod p^n \mathbb A.$$
    On commence par prouver: $\quad \boxed{ \forall a \in \Z,\quad \displaystyle \prod_{j=1}^{p-1} (1 +\dfrac {ap}j) \equiv 1 \mod p^3 \mathbb A. \quad (\bigstar) }$
    $ \displaystyle \prod _{j=1}^{p-1}( 1+ \dfrac {ap}j) \equiv 1 +ap \sum _{j=1}^{p-1}\dfrac 1j + a^2p^2 \sum _{1\leqslant i<j\leqslant p-1} \dfrac1 {ij} \mod p^3 \mathbb A.\quad$ D'une part, le théorème de Wolstenholme dit que $\displaystyle \sum _{j=1}^{p-1} \dfrac 1j \equiv 0 \mod p^2 \mathbb A$
    et d'autre part :$ \:\: \displaystyle\sum_{1\leqslant i<j \leqslant p-1} \dfrac 1{ij} \equiv \sum _{1 \leqslant i<j \leqslant p-1} \hat i \hat j \equiv \sum _{1\leqslant i<j \leqslant p-1} ij \equiv \dfrac{p(p-1)(p-2)(3p-1)}{24} \equiv 0 \mod p\mathbb A.$
    De ces deux congruences, on déduit alors immédiatement $(\star)\quad\square$

    Revenons à notre problème:$\:\: \displaystyle \binom {pm}{pn} = \binom mn \prod _{k=0}^{n-1} \prod _{j=1}^{p-1}\dfrac {(k+m-n)p +j}{kp+j} =\binom mn \prod_{k=0}^{n-1} \prod_{j=1}^{p-1} \dfrac {(1+\frac {a_kp}j)}{(1 + \frac {b_kp}j)}\quad$ où $a_k,b_k \in \Z.$
    En injectant $(\bigstar)$ dans cette égalité, on parvient à : $\displaystyle \quad \boxed{\displaystyle \binom {pm}{pn} \equiv \binom mn \mod p^3.}$
  • Merci à tous !
    @Chaurien, j'ai lu les deux pages internet citées. La preuve par dénombrement avec une action de groupe est élégante :-). En fait j'avais vu l'exercice de ce fil dans un oral (d'ENS si je me souviens bien) et la première question était justement le théorème de Lucas (l'exercice du fil étant la deuxième question).
    P.S.: Je n’avais, avant de poster mon deuxième message, pas pris le temps de me poser pour vérifier comme il se doit la preuve d’YvesM (d'où ma réponse fausse à Chaurien). J’aurais dû, désolé.
  • Moi je procède comme suit. Pour $m\in \mathbb{N}$ et $n\in \mathbb{N}^{\ast }$ soit $(m)_{n}=m(m-1)...(m-n+1)$ ($n$ facteurs), et de plus $(m)_{0}=1$. C'est une notation à la mode pour les bons vieux arrangements, d'où classiquement : $\displaystyle \binom {m} {n}=\frac {(m)_n}{n!}$.

    Et puis, c'est du calcul.
    Si $p\in \mathbb{N}^{\ast }$ et $a\in \mathbb{N}^{\ast }$, alors : $\displaystyle (ap)!=a!p^{a}\overset{a}{\underset{k=1}{\prod }}(kp-1)_{p-1}=a!(p!)^{a}\overset{a}{\underset{k=1}{\prod }}\binom{kp-1}{p-1}$.

    Et si $a,b,p$ sont des entiers tels que : $a>b \ge1$ et $p \ge 1$, alors :
    $\displaystyle \binom{ap}{bp}=\frac{(ap)!}{(bp)!((a-b)p)!} = \frac{a!}{b!(a-b)!}\cdot $$ \frac {\displaystyle \overset{a}{\underset{k=1}{\prod }}\binom{kp-1}{p-1}} {\displaystyle \overset{b}{\underset{k=1}{\prod }}\binom{kp-1}{p-1} \displaystyle \overset{a-b}{\underset{k=1}{\prod }}\binom{kp-1}{p-1}}$.

    Maintenant, autour du théorème de Wolstenholme, on démontre que pour $k\in \mathbb{N}^{\ast }$ et $p$ entier premier, $p\geq 5$, on a : $\binom{kp-1}{p-1}\equiv 1$$\pmod{p^3}$. J'y reviendrai si besoin est. Ceci achève la preuve.

    Bonne soirée.
    Fr. Ch.
    09/11/2019
  • $\bullet $ Voici le lien entre le théorème de Wolstenholme sous sa forme classique bien connue et le résultat que j'utilise dans mon précédent message.
    Je reprends les notations dudit message.
    Les relations très connues entre les coefficients et les racines d'un polynôme à une indéterminée conduisent à :
    $\displaystyle (x-1)_{p-1}=(x-1)(x-2)...(x-p+1)=\overset{p-1}{\underset{\ell =1}{\prod }}(x-\ell )=x^{p-1}+\overset{p-1}{\underset{k=1}{\sum }}(-1)^{k}\sigma _{k}x^{p-1-k}$,
    où : $\displaystyle \sigma _{k}=\underset{1\leq \ell _{1}<\ell _{2}<...<\ell _{k}\leq p-1}{\sum }\ell _{1}\ell _{2}...\ell _{k}$ (fonction symétrique élémentaires de $1,2,...,p-1$, avec $\binom{p-1}{k}$ termes).

    $\bullet $ Supposons $\displaystyle k\in \mathbb{N}^{\ast }$ et $p$ entier impair, $p\geq 5$, alors : $(p-1)!\binom{kp-1}{p-1}=(kp-1)_{p-1}=...$ $~~~~~~~~~~~~~~~~\displaystyle =(kp)^{p-1}+(~\overset{p-4}{\underset{k=1}{\sum }}(-1)^{k}\sigma
    _{k}(kp)^{p-1-k}~)+\sigma _{p-3}(kp)^{2}-\sigma _{p-2}kp+\sigma _{p-1}$.
    Comme $\displaystyle \sigma _{p-1}=(p-1)!$, il en résulte : $(p-1)!~(~\binom{kp-1}{p-1}-1~)\equiv k^{2}\sigma _{p-3}p^{2}-k\sigma _{p-2}p \pmod{p^3} $.

    $\bullet $ Le théorème de Wolstenholme dit que si $p$ est un entier naturel premier, $p\geq 5$, alors le numérateur du rationnel $\displaystyle H_{p-1}=\frac{1}{1}+\frac{1}{2}+...+\frac{1}{p-1}=\overset{p-1}{\underset{i=1}{\sum }}\frac{1}{i}$ est divisible par $p^{2}$, d'où il n'est pas difficile de déduire que le numérateur de $\displaystyle \frac{1}{1^{2}}+\frac{1}{2^{2}}+...+\frac{1}{(p-1)^{2}}=\overset{p-1}{\underset{i=1}{\sum }}\frac{1}{i^{2}}$ est divisible par $p$, ainsi que le numérateur de $\displaystyle \underset{1\leq i<j\leq p-1}{\sum }\frac{1}{ij}$.
    Or $\displaystyle \overset{p-1}{\underset{i=1}{\sum }}\frac{1}{i}=\frac{\sigma _{p-2}}{(p-1)!}$ et $\displaystyle \underset{1\leq i<j\leq p-1}{\sum }\frac{1}{ij}=\frac{\sigma _{p-3}}{(p-1)!}$, ce qui prouve que $\sigma _{p-2}$ est multiple de $p^{2}$ et que $\sigma _{p-3}$ est multiple de $p$.
    On en déduit finalement : $\displaystyle \binom{kp-1}{p-1}\equiv 1\pmod{p^3}$.

    $\bullet $ Maintenant il faudrait revenir sur les démonstrations du théorème de Wolstenholme, comme celle qu'on trouve dans le Hardy & Wright, laquelle m'a longtemps semblé mystérieuse, car ces congruences dans le corps $\mathbb Q$ ne me disaient rien qui vaille. En fait il s'agissait de congruences dans l'anneau local $\mathbb A_p$ défini par LOU16. J'en ai bricolé naguère une démonstration qui n'utilise pas cet outil, ni $\mathbb Z/p \mathbb Z$.

    Bonne soirée.
    Fr. Ch.
    11/11/2019
    [small]Saluons la Fête de la Victoire, honneur au brigadier Ronan Pointeau tué à l'ennemi islamiste au Mali, et honte aux calomniateurs de l'armée française.[/small]
  • Bonsoir Chaurien,

    Ce sujet a été traité il n'y a pas longtemps ici:
    Wolstenhome

    Cidrolin a proposé une page où tu trouveras une preuve assez simple à comprendre. J'en ai proposé une autre.

    Al-Kashi
  • @ « Al-Kashi »
    Merci de me proposer une preuve « assez simple à comprendre », des fois que j'aurais un souci de ce côté.
    Je trouve que dans plusieurs de ces preuves il y a des confusions entre l'inverse dans $\mathbb Q$ et l'inverse dans $\mathbb Z /p \mathbb Z$, ce qui rend pour moi ces preuves peu claires, peu convaincantes, peu reproductibles. Ce n'est pas ma « compréhension » qui est en cause, c'est la mauvaise qualité de ces preuves.
    Merci pour tes références, mais j'ai aussi une bibliographie sur la question, que je donnerai peut-être si j'ai le temps et si ça vaut la peine.
    En 1999 j'ai rédigé une démonstration qui comme j'ai dit dans mon précédent message ne sort pas de $\mathbb Z$.
    Elle est aussi « assez simple à comprendre » et je vais sans doute la donner ici car ton message m'y incite.
  • Vingt ans après, voici une démonstration que j'ai bricolée en 1999, qui ne sort pas de $\mathbb Z$.

    THÉORÈME DE WOLSTENHOLME (1862)
    $\bullet $ Soit un nombre entier premier $p\geq 5$, et $H_{p-1}=\frac{1}{1}+\frac{1}{2}+...+\frac{1}{p-1}=\frac{a}{b}$, $a\in \mathbb{N}^{\ast }$, $b\in \mathbb{N}^{\ast }$. Démontrer que $a$ est divisible par $p^{2}$.

    $\bullet $ On a : $\displaystyle 2H_{p-1}=\overset{p-1}{\underset{k=1}{\sum }}(\frac{1}{k}+\frac{1}{p-k})=p\overset{p-1}{\underset{k=1}{\sum }}\frac{1}{k(p-k)}$. Avec cette remarque, Cauchy avait déjà prouvé en 1840 que $a$ est
    divisible par $p$.

    $\bullet $ Chaque entier $k\in \{1,2,...,p-1\}$ est premier avec $p$, et d'après Bachet-Bézout il existe donc $u_{k}\in \{1,2,...,p-1\}$ tel que : $ku_{k}\equiv 1 \pmod p$. En conséquence : $k^{2}u_{k}^{2}\equiv 1 \pmod p$, soit : $k^{2}u_{k}^{2}=1+pv_{k}$, $v_{k}\in \mathbb{Z}$ (en fait $v_{k}\in \mathbb{N}$ mais peu importe).
    Il en résulte : $\frac{1}{k(p-k)}+u_{k}^{2}=\frac{1+kpu_{k}^{2}-k^{2}u_{k}^{2}}{k(p-k)}=p\frac{ku_{k}^{2}-v_{k}}{k(p-k)}=p%
    \frac{w_{k}}{k(p-k)}$, avec $w_{k}\in \mathbb{Z}$.
    D'où : $\frac{1}{k(p-k)}=p\frac{w_{k}}{k(p-k)}-u_{k}^{2}$.

    $\bullet $ Lorsque $k$ décrit l'ensemble $\{1,2,...,p-1\}$ les entiers $u_{k}$ sont distincts, d'où : $\{u_{1},u_{2},...,u_{p-1}\}=\{1,2,...,p-1\}$, et il s'ensuit :
    $\displaystyle \overset{p-1}{\underset{k=1}{\sum }}u_{k}^{2}=\overset{p-1}{\underset{k=1}{\sum }}k^{2}=\frac{p(p-1)(2p-1)}{6}$.
    Le nombre premier $p\geq 5$ étant premier avec $6$, cet entier $\frac{p(p-1)(2p-1)}{6}$ est multiple de $p$, soit : $\displaystyle \overset{p-1}{\underset{k=1}{\sum }}u_{k}^{2}=pq$, $q\in \mathbb{Z}$.

    $\bullet $ On en déduit au final :
    $\displaystyle 2\frac{a}{b}=2H_{p-1}=p\overset{p-1}{\underset{k=1}{\sum }}\frac{1}{k(p-k)}=p\overset{p-1}{\underset{k=1}{\sum }}(p\frac{w_{k}}{k(p-k)}-u_{k}^{2})=p^{2}\overset{p-1}{\underset{k=1}{\sum }}\frac{w_{k}}{k(p-k)}-p^{2}q$,
    et par suite :
    $\displaystyle 2a(p-1)!=bp^{2}(~\overset{p-1}{\underset{k=1}{\sum }}w_{k}\frac{(p-1)!}{k(p-k)}-(p-1)!q)=bp^{2}C$, $C\in \mathbb{Z}$.
    D'après Gauss, il apparaît bien que $p^{2}$ est diviseur de $a$.

    $\bullet $ Cette démonstration ne sera peut-être pas du goût de tout le monde, mais elle a le mérite de démontrer la propriété en question par des voies purement élémentaires.

    Bonne journée.
    Fr. Ch.
    12/11/2019
  • Le charabia de Tonm ne me convainc pas.
    Ne répondons pas à de tels messages hors-culturels.

    [Un peu de tolérance envers les intervenants dont le français n'est pas la langue maternelle. Merci. AD]
  • Dans la série des belles propriétés arithmétiques des coefficients binomiaux, en voici encore une, datant si je ne me trompe de 1950.
    Si $n\in \mathbb{N}$, si $p$ est un nombre entier naturel premier, alors : $\binom{n}{p}\equiv \left\lfloor \frac{n}{p}\right\rfloor \pmod p$.
    Bonne soirée.
    Fr. Ch.
  • Joli. Cette congruence porte-t-elle un nom ?
  • Rebonjour, il y a un truc dans ces congruences qu'ils marchent si $p$ est premier, et la plupart des cas on sort parce que on peut travailler $\pmod{p}$ les inverses où les dénominateurs sont des facteurs de la forme $\prod (p-i)=b$, donc une fraction entière $\dfrac{a}{b}=a.b^{-1} \pmod{p}$ ou on remplace directement le $b$ par son représentant dans $\mathbb{Z}/{p\mathbb{Z}}$.
    Ça me perd.

    Cordialement.
  • @ Poirot
    Je ne sais pas grand'chose sur cette jolie congruence $ \binom{n}{p}\equiv \left\lfloor \frac{n}{p}\right\rfloor \pmod p $, sinon qu'elle a été posée comme problème dans l'American mathematical Monthly en 1948, solution en 1950, et la réciproque comme nouveau problème en 1956, et généralisation à $p^\alpha$ au lieu de $p$.
    Bonne soirée.
    Fr. Ch.
Connectez-vous ou Inscrivez-vous pour répondre.